Đến nội dung

vo thanh van

vo thanh van

Đăng ký: 07-10-2006
Offline Đăng nhập: 16-04-2018 - 23:27
****-

#186312 BẤT ĐẲNG THỨC SCHUR VÀ PHƯƠNG PHÁP ĐỔI BIẾN P,Q,R

Gửi bởi vo thanh van trong 03-06-2008 - 20:35

Ví dụ 12: Võ Quốc Bá Cẩn
Cho $a,b,c\ge 0$ thỏa mãn $a+b+c=3$.Chứng minh rằng

$\dfrac{a}{2b+1}+\dfrac{a}{2b+1}+\dfrac{a}{2b+1}\le \dfrac{1}{abc}$
Lời giải:Nguyễn Thúc Vũ Hoàng

Quy đồng,rút gọn,đổi biến BDT thành $p,q,r$ và xét hàm theo $r$,ta cần chứng minh
$f®=4r^2+(4q-17)r-2q^2+4q+7\ge 0$
Áp dụng BDT Schur,ta có
$\dfrac{12q-27}{9}\le r \le\dfrac{q}{3}$
.TH1:$4q+8r-17\ge 0$,khi đó $f^{'}®=4q+8r-17\ge 0\Rightarrow f®\ge f(\dfrac{12q-27}{9})=\dfrac{94(3-q)^2}{9}\ge 0$
.TH2:$4q+8r-17\le 0$,khi đó $f^{'}®=4q+8r-17\le 0\Rightarrow f®\ge f(\dfrac{q}{3})=\dfrac{(3-q)(2q+21)}{9}\ge 0$ với $\forall q\le 3$
Đẳng thức xảy ra khi $a=b=c=1$.
Ví dụ 13:Vasc
Cho a,b,c là các số thực không âm.Chứng minh rằng:

$a^4(b+c)+b^4(c+a)+c^4(a+b)\le \dfrac{1}{12}(a+b+c)^5$
Lời giải:

Chuẩn hóa $p=1$,ta có bất đẳng thức:
$(1-3q)q+5qr-r\le \dfrac{1}{12}$
Đến đây ta sử dụng một thủ thuật khi dùng bất đẳng thức Schur ,đó là chia trường hợp để giải quyết:
Nếu $q\le \dfrac{1}{5}$ thì ta có $5qr-r\le 0$ và $(1-3q)q=\dfrac{1}{3}(1-3q)3q\le \dfrac{1}{3}(\dfrac{1-3q+3q}{2})^2=\dfrac{1}{12}$ (dpcm)
Nếu $q>\dfrac{1}{5}$ ta đưa bất đẳng thức cần chứng minh thành một hàm theo $q$: $f(q)=(1-3q)q+5qr-r$
Xét $f^{'}(q)=1-6q+5r$
Vì $\dfrac{1}{5}>q\ge 9r$ nên $f^{'}(q)<0$ suy ra $f(q)< f(\dfrac{1}{5})=\dfrac{2}{15}<\dfrac{1}{12}$
Vậy bất đẳng thức được chứng minh.
Đẳng thức xảy ra khi $a=0,b=\dfrac{3+\sqrt{3}}{6},c=\dfrac{3-\sqrt{3}}{6}$ và các hoán vị


#186310 BẤT ĐẲNG THỨC SCHUR VÀ PHƯƠNG PHÁP ĐỔI BIẾN P,Q,R

Gửi bởi vo thanh van trong 03-06-2008 - 20:32

3.2: Phương pháp đổi biến p,q,r:
Ví dụ 10: Phạm Kim Hùng

Cho a,b,c là các số thực không âm thỏa mãn a+b+c=3.Chứng minh rằng:
$\dfrac {a^2b}{4 - bc} + \dfrac {b^2c}{4 - ca} + \dfrac {c^2a}{4 - ab} \le 1$
Lời giải:

Quy đồng mẫu số rồi khai triển,ta cần chứng minh:
$4 - \sum a^2b \ge \sum \dfrac {a^2b^2c}{4 - bc}$
Sử dụng bất đẳng thức quen biết $4 - \sum a^2b \ge abc$,ta có:
$1 \ge \sum \dfrac {ab}{4 - bc}$
$\Leftrightarrow64 - 32\sum ab + 8\sum a^2bc + 4\sum a^2b^2 \ge abc(\sum a^2b + abc)$
Tiếp tục sử dụng bất đẳng thức trên,ta cần chứng minh:
$64 - 32\sum ab + 8\sum a^2bc + 4\sum a^2b^2 \ge 4abc$
hay $16 - 8q + q^2 - r \ge 0$
với $q = ab + bc + ca,r = abc$. Áp dụng BDT AM-GM,ta có$q^2 \ge 9r$ nên cần chứng minh:
$16 - 8q + q^2 - \dfrac {q^2}{9} \ge 0 \Leftrightarrow(q - 3)(q - 6) \ge 0$
Bắt đẳng thức cuối hiển nhiên đúng nên ta có đpcm.
Đẳng thức xảy ra khi $a = b = c = 1$ hoặc $a = 2,b = 1,c = 0$ và các hoán vị.
Ví dụ 11: Dương Đức Lâm
Cho a,b,c > 0.Chứng minh rằng

$\dfrac1{a} + \dfrac1{b} + \dfrac1{c}\ge \dfrac {3a}{a^2 + 2bc} + \dfrac {3b}{b^2 + 2ca} + \dfrac {3c}{c^2 + 2ab}$
Lời giải:Võ Quốc Bá Cẩn

Đặt $a = \dfrac {1}{a},b= \dfrac {1}{b},c = \dfrac {1}{c}$ bất đẳng thức cần chứng minh tương đương với
$\sum a \ge 3abc\sum \dfrac {1}{2a^2 + bc}$
$\Leftrightarrow \sum \dfrac {a(a^2 - bc)}{2a^2 + bc} \ge 0$
$\Leftrightarrow 3\sum \dfrac {a^3}{2a^2 + bc} \ge \sum a$
Áp dụng BDT Cauchy-Schwarz,ta có
$\sum \dfrac {a^3}{2a^2 + bc} \ge \dfrac {(\sum a^2)^2}{2\sum a^3 + 3abc}$
Đến đây ta cần chứng minh
$3(\sum a^2)^2 \ge (\sum a)(2\sum a^3 + 3abc)$
Giả sử $a + b + c = 1$ chuyển về dạng $p,q,r$,BDT trở thành
$3(1 - 2q)^2 \ge 2 - 6q + 9r$
Từ $q^2 \ge 3r$ ta có:
$3(1 - 2q)^2 \ge 2 - 6q + 3q^2$
$\Leftrightarrow 3 - 12q + 12q^2 \ge 2 - 6q + 3q^2$
$\Leftrightarrow (1 - 3q)^2 \ge 0$


#186309 BẤT ĐẲNG THỨC SCHUR VÀ PHƯƠNG PHÁP ĐỔI BIẾN P,Q,R

Gửi bởi vo thanh van trong 03-06-2008 - 20:31

Ví dụ 8: Phạm Kim Hùng

Cho a,b,c là các số thực dương thỏa mãn $a^2+b^2+c^2=3$.Chứng minh rằng:
$\dfrac{1}{2-a}+\dfrac{1}{2-b}+\dfrac{1}{2-c}\geq 3$
Lời giải:

Quy đồng,rút gọn và đổi biến theo p,q,r,bất đẳng thức cần chứng minh tương đương với
$8p+3r\geq 12+5q$
Áp dụng BDT Schur,ta có:
$3r\geq \dfrac{p(4q-p^2)}{3}=\dfrac{p(2q-3)}{3}$
Từ giả thiết $p^2-2q=3 \Rightarrow q=\dfrac{p^2-3}{2}$
Thay 2 điều trên vào bất đẳng thức cần chứng minh,ta có:
$8p+\dfrac{p(p^2-6)}{3}\geq 12+\dfrac{5(p^2-3)}{2}$
$\Leftrightarrow (2p-3)(p-3)^2\geq 0$
Bất đẳng thức cuối đúng nên ta có đpcm.
Ví dụ 9:CRUX

Cho a,b,c là các số thực không âm thỏa mãn $a+b+c=3$.Chứng minh rằng:
$\dfrac{1}{9-ab}+\dfrac{1}{9-bc}+\dfrac{1}{9-ca}\le \dfrac{3}{8}$

* Bài này đã được anh Hùng sử dụng cho phần BDT TRê-bư-sép trong cuốn Sáng tạo BDT,tuy nhiên bây giờ các bạn sẽ được thấy một lời giải với BDT Schur và phương pháp đổi biến p,q,r rất tự nhiên.

Lời giải:

Biến đổi bất đẳng thức cần chứng minh và chuyển về dạng $p,q,r$,ta có:
$8(243-18p+3r) \leq 3(729-81q+27r-r^2)$
$\Leftrightarrow 243-99q+57r-3r^2\ge 0$
Theo BDT AM-GM thì $3=3(\dfrac{a+b+c}{3})^6\geq 3(abc)^2=r^2$
Theo BDT Schur,ta có:
$r\geq \dfrac{p(4q-p^2)}{3}=\dfrac{4q-9}{3} \Rightarrow 57r\ge 19(4q-9)$
Nên ta cần chứng minh:
$72-23q-3r^2\ge 0\Leftrightarrow 3(1-r^2)+23(3-q)\ge 0$
Vậy BDT được chứng minh.


#186308 BẤT ĐẲNG THỨC SCHUR VÀ PHƯƠNG PHÁP ĐỔI BIẾN P,Q,R

Gửi bởi vo thanh van trong 03-06-2008 - 20:30

Ví dụ 5:Moldova TST 2005:

Chứng minh rằng nếu a,b,c là các số thực dương và $a^4+b^4+c^4=3$ thì:
$\dfrac{1}{4-ab}+\dfrac{1}{4-bc}+\dfrac{1}{4-ca}\leq 1$
Lời giải:

Quy đồng mẫu số rồi khai triển,ta cần chứng minh:
$49-8(ab+bc+ca)+(a+b+c)abc\le 64-16(ab+bc+ca)+4(a+b+c)abc-a^2b^2c^2$
$\Leftrightarrow 16+3(a+b+c)abc\geq a^2b^2c^2+8(ab+bc+ca)$
Áp dụng bất đẳng thức Schur và giả thiết $a^4+b^4+c^4=3$,ta có:
$(a^3+b^3+c^3+3abc)(a+b+c)\geq (ab(a+b)+bc(b+c)+ca(c+a))(a+b+c)$
$\Leftrightarrow 3+3abc(a+b+c)\geq (ab+bc)^2+(bc+ca)^2+(ca+ab)^2$
Áp dụng BDT AM-GM,ta có:
$(ab+bc)^2+(bc+ca)^2+(ca+ab)^2+12\geq 8(ab+bc+ca)$
$\Rightarrow 15+3abc(a+b+c)\geq 8(ab+bc+ca)$
Mặt khác ta lại có:$1\geq a^2b^2c^2$
Vậy ta có đpcm.
Ví dụ 6:Vasile Cirtoaje:

Cho $a,b,c$ là các số thực không âm thỏa mãn $ab+bc+ca=3$.Chứng minh rằng:$a^3+b^3+c^3+7abc\geq 10$
Lời giải:

Áp dụng BDT Schur,ta có:
$a^3+b^3+c^3+3abc\geq ab(a+b)+bc(b+c)+ca(c+a)$
$\Leftrightarrow a^3+b^3+c^3+6abc\geq (ab+bc+ca)(a+b+c)=pq=3p$
và $r\geq \dfrac{p(4q-p^2)}{9}=\dfrac{p(12-p^2)}{9}$
Ta cần chứng minh:
$3p+\dfrac{p(12-p^2)}{9}\geq 10$
$\Leftrightarrow \dfrac{(p-3)[(16-p^2)+3(4-p)+2]}{9}\geq 0$
Bất đẳng thức cuối hiển nhiên đúng nên ta có đpcm.
Đẳng thức xảy ra khi $a=b=c=1$.
Ví dụ 7:Võ Thành Văn:

Cho $a,b,c>0,a+b+c=3$.Chứng minh rằng:
$3+\dfrac{12}{abc}\geq 5(\dfrac{1}{a}+\dfrac{1}{b}+\dfrac{1}{c})$
Lời giải:

Đổi biến theo p,q,r,bât đẳng thức cần chứng minh được viết lại như sau:
$3r+12\geq 5q $($\star$)
Mặt khác,theo BDT Schur,ta có:
$3r\geq \dfrac{3p(4q-p^2)}{9}=4q-9$
$\Rightarrow (\star)\Leftrightarrow 4q-9+12\geq 5q\Leftrightarrow q\le 3(dung)$
Vậy ta có đpcm


#186307 BẤT ĐẲNG THỨC SCHUR VÀ PHƯƠNG PHÁP ĐỔI BIẾN P,Q,R

Gửi bởi vo thanh van trong 03-06-2008 - 20:27

Ví dụ 3:VMO 2002-Trần Nam Dũng
Chứng minh rằng với mọi $a,b,c\geq 0$,ta có:
$2(a^2+b^2+c^2)+abc+8\geq 5(a+b+c)$
Lời giải:
Sử dụng bất đẳng thức AM-GM,ta có:
$12(a^2+b^2+c^2)+6abc+48-30(a+b+c)$
$=12(a^2+b^2+c^2)+3(2abc+1)+45-5.2.3(a+b+c)$
$\geq12(a^2+b^2+c^2)+9\sqrt[3]{a^2b^2c^2}+45-5.((a+b+c)^2+9)$
$=7(a^2+b^2+c^2)+\dfrac{9abc}{\sqrt[3]{abc}-10(ab+bc+ca)$
$\geq 7(a^2+b^2+c^2)+\dfrac{27}{a+b+c}-10(ab+bc+ca)
$
Mặt khác sử dụng bất đẳng thức Schur,
$\dfrac{9}{a+b+c}\geq 4(ab+bc+ca)-(a+b+c)^2=2(ab+bc+ca)-(a^2+b^2+c^2)$
Do đó
$7(a^2+b^2+c^2)+\dfrac{27}{a+b+c}-10(ab+bc+ca)$
$\geq 7(a^2+b^2+c^2)+6(ab+bc+ca)-3(a^2+b^2+c^2)-10(ab+bc+ca)=4(a^2+b^2+c^2-ab-bc-ca)\geq 0$
Bất đẳng thức được chứng minh.
Ví dụ 4:Arqady

Cho a,b,c là các số không âm,trong đó không có 2 số nào đồng thời bằng 0.Chứng minh rằng:
$\dfrac {a}{b^{3} + c^{3}} + \dfrac {b}{a^{3} + c^{3}} + \dfrac {c}{a^{3} + b^{3}}\geq\dfrac {18}{5(a^{2} + b^{2} + c^{2}) - ab - ac - bc}$
Lời giải:

Bất đẳng thức cần chứng minh tương đương với
$\sum \dfrac {a(a + b + c)}{b^3 + c^3} \ge \dfrac {18(a + b + c)}{5(a^2 + b^2 + c^2) - ab - bc - ca}$
$\Leftrightarrow \sum \dfrac {a^2}{b^3 + c^3} + \dfrac {a}{b^2 + c^2 - bc} \ge \dfrac {18(a + b + c)}{5(a^2 + b^2 + c^2) - ab - bc - ca}$
Áp dụng bất đẳng thức Cauchy-Schwarz,ta có:
$i) \sum \dfrac {a^2}{b^3 + c^3} \ge \dfrac {(a^2 + b^2 + c^2)^2}{\sum a^2(b^3 + c^3)}$
$ii) \sum \dfrac {a}{b^2 + c^2 - bc} \ge \dfrac {(a + b + c)^2}{\sum a(b^2 + c^2 - bc)}$
Áp dụng 2 bất đẳng thức trên,ta có:
$\dfrac {(a^2 + b^2 + c^2)^2}{\sum a^2(b^3 + c^3)} + \dfrac {(a + b + c)^2}{\sum a(b^2 + c^2 - bc)} \ge \dfrac {18(a + b + c)}{5(a^2 + b^2 + c^2) - ab - bc - ca}$
Giả sử $a + b + c = 1$ và đặt $ab + bc + ca = q,abc = r \Rightarrow r \ge \max \left{ 0,\dfrac {(4q - 1)(1 - q)}{6}\right }$.
Ta cần chứng minh
$\dfrac {(1 - 2q)^2}{q^2 - (q + 2)r} + \dfrac {1}{q - 6r} \ge \dfrac {18}{5 - 11q}$
Bất đẳng thức cuối dễ dàng chứng minh bằng cách xét 2 trường hợp:$1 \ge 4q$ và $4q \ge 1$ :lol:
Đẳng thức xảy ra khi $a = b = c$ và $a = b,c = 0$.


#186306 BẤT ĐẲNG THỨC SCHUR VÀ PHƯƠNG PHÁP ĐỔI BIẾN P,Q,R

Gửi bởi vo thanh van trong 03-06-2008 - 20:25

III-VÍ DỤ MINH HỌA
3.1:Bất đẳng thức Schur:
Ví dụ 1:Võ Thành Văn:

Cho a,b,c là các số thực không âm.Chứng minh rằng:
$\sqrt{\dfrac{(a+b)^3}{8ab(4a+4b+c)}}+\sqrt{\dfrac{(b+c)^3}{8bc(4b+4c+a)}}+\sqrt{\dfrac{(c+a)^3}{8ca(4c+4a+b)}}\geq 1$

Lời giải:


Đặt $P=\sqrt{\dfrac{(a+b)^3}{8ab(4a+4b+c)}}+\sqrt{\dfrac{(b+c)^3}{8bc(4b+4c+a)}}+\sqrt{\dfrac{(c+a)^3}{8ca(4c+4a+b)}}$
$Q=8ab(4a+4b+c)+8bc(4b+4c+a)+8ca(4c+4a+b)$
$=\sum 32ab(a+b)+24abc$
$=32(a+b+c)(ab+bc+ca)-72abc$
Áp dụng BDT Holder,ta có:
$P^2.Q\geq 8(a+b+c)^3$
Ta cần chứng minh:
$8(a+b+c)^3\geq Q\Leftrightarrow8(a+b+c)^3\geq 32(a+b+c)(ab+bc+ca)-72abc$
$\Leftrightarrow (a+b+c)^3\geq 4(a+b+c)(ab+bc+ca)-9abc$(đúng theo BDT Schur)
Vậy ta có đpcm.
Ví dụ 2:APMO 2004:

Cho 3 số thực dương $a,b,c$.Chứng minh rằng:
$(a^2+2)(b^2+2)(c^2+2)\geq 9(ab+bc+ca)$

Lời giải


Lời giải 1:Khai triển bất đẳng thức trên,ta cần chứng minh:
$a^2b^2c^2+2(a^2b^2+b^2c^2+c^2^2)+4(a^2+b^2+c^2)+8 \geq 9(ab+bc+ca)$
Ta có:$i.a^2+b^2+c^2\geq ab+bc+ca$
$ii.(a^2b^2+1)+(b^2c^2+1)+(c^2a^2+1)\geq 2(ab+bc+ca)$
$iii.a^2b^2c^2+1+1\geq 3\sqrt[3]{a^2b^2c^2}\geq \dfrac{9abc}{a+b+c}\geq 4(ab+bc+ca)-(a+b+c)^2$ (theo BDT Schur)
Áp dụng các BDT trên,ta có:
$(a^2b^2c^2+2)+2(a^2b^2+b^2c^2+c^2a^2+3)+4(a^2+b^2+c^2)\geq 2(ab+bc+ca)+4(ab+bc+ca)+3(a^2+b^2+c^2)\geq 9(ab+bc+ca) (dpcm)$
Lời giải 2:
Sử dụng bất đẳng thức AM-GM,ta có:
$(a^2+2)(b^2+2)(c^2+2)-9(ab+bc+ca)$
$=4(a^2+b^2+c^2)+2((a^2b^2+1)+(b^2c^2+1)+(c^2a^2+1))+(a^2b^2c^2+1)+1-9(ab+bc+ca)$
$\geq 4(a^2+b^2+c^2)+4(ab+bc+ca)+2abc+1-9(ab+bc+ca)$
$\geq a^2+b^2+c^2+2abc+1-2(ab+bc+ca)$
Bất đẳng thức cuối đã rất quen thuộc,ta có đpcm.


#186305 BẤT ĐẲNG THỨC SCHUR VÀ PHƯƠNG PHÁP ĐỔI BIẾN P,Q,R

Gửi bởi vo thanh van trong 03-06-2008 - 20:24

BẤT ĐẲNG THỨC SCHUR VÀ PHƯƠNG PHÁP ĐỔI BIẾN P,Q,R
Võ Thành Văn
Lớp 11 Toán-Khối chuyên THPT-ĐHKH Huế


*LỜI MỞ ĐẦU:
Như các bạn đã biết,bất đẳng thức Schur là một bất đẳng thức mạnh và có nhiều ứng dụng,tuy nhiên nó vẫn còn khá xa lạ với nhiều bạn học sinh THCS cũng như THPT.Qua bài viết này,tôi muốn cũng cấp thêm cho các bạn một kĩ thuật để sử dụng tốt BDT Schur,đó là kết hợp với phương pháp đổi biến $p,q,r$.
Trước hết tôi xin nhắc lại về bất đẳng thức Schur và phương pháp đổi biến $p,q,r$.
I-BẤT ĐẲNG THỨC SCHUR:
Với các số thực dương a,b,c và $k\in R^+$ bất kì ta luôn có
$a^k(a-b)(a-c)+b^k(b-c)(b-a)+c^k(c-a)(c-b)\geq 0$
Hai trường hợp quen thuộc được sử dụng nhiều là k=1 và k=2:
$a(a-b)(a-c)+b(b-c)(b-a)+c(c-a)(c-b)\geq 0 (i)$
$a^2(a-b)(a-c)+b^2(b-c)(b-a)+c^2(c-a)(c-b)\geq 0 (ii)$
II-PHƯƠNG PHÁP ĐỔI BIẾN P,Q,R:
Đối với một số bài bất đẳng thức thuần nhất đối xứng có các biến không âm thì ta có thể đổi biến lại như sau:
Đặt $p=a+b+c,q=ab+bc+ca,r=abc$
Ta có một số đẳng thức sau:
.$ab(a+b)+bc(b+c)+ca(c+a)=pq-3r$
$.(a+b)(b+c)(c+a)=pq-r$
$.ab(a^2+b^2)+bc^(b^2+c^2)+ca(c^2+a^2)=p^2q-2q^2-pr$
$.(a+b)(a+c)+(b+c)(b+a)+(c+a)(c+b)=p^2+q$
$.a^2+b^2+c^2=p^2-2q$
$.a^3+b^3+c^3=p^3-3pq+3r$
$.a^4+b^4+c^4=p^4-4p^2q+2q^2+4pr$
$.a^2b^2+b^2c^2+c^2a^2=q^2-2pr$
$.a^3b^3+b^3c^3+c^3a^3=q^3-3pqr+3r^2$
$.a^4b^4+b^4c^4+c^4a^4=q^4-4pq^2r+2p^2r^2+4qr^2$
Đặt $L=p^2q^2+18pqr-27r^2-4q^3-4p^3r$
Khi đó $a^2b+b^2c+c^2a=\dfrac{pq-3r+/- \sqrt{L}}{2}$
$(a-b)(b-c)(c-a)=\sqrt{L}$
Có thể thấy ngay lợi ích của phương pháp này là mối ràng buộc giữa các biến p,q,r mà các biến a,b,c ban đầu không có như:
$.p^2\geq 3q$
$.p^3\geq 27r$
$.q^2\geq 3pr$
$.pq\geq 9r$
$.2p^3+9r\geq 7pq$
$.p^2q+3pr\geq 4q^2$
$.p^4+4q^2+6pr\geq 5p^2q$
Những kết quả trên đây chắc chắn là chưa đủ,các bạn có thể phát triển thêm nhiều đẳng thức,bất đẳng thức liên hệ giữa 3 biến p,q,r.Và điều quan trọng mà tôi muốn nói đến là từ bất đẳng thức $(i)$ và $(ii)$,ta có:
$r\geq \dfrac{p(4q-p^2)}{9}$ (từ $(i)$)
$r\geq \dfrac{(4q-p^2)(p^2-q)}{6p}$ (từ $(ii)$)
Tuy nhiên trong một số trường hợp thì có thể các đại lượng $4q-p^2$ có thể nhận giá trị âm lẫn giá trị dương nên ta thường sử dụng
$.r \ge \max \left( 0,\dfrac {p(4q-p^2)}{4}\right )$
$.r \ge \max \left( 0,\dfrac {(4q - p^2)(p^2 - q)}{6p}\right )$
Có lẽ đến đây các bạn đã hiểu được phần nào về bất đẳng thức Schur và phương pháp đổi biến p,q,r.Sau đây là một số ví dụ minh họa,nhưng trước hết,các bạn hãy tập làm thử rồi xem đáp án sau:


#186243 Giải toán bằng phương pháp tọa độ

Gửi bởi vo thanh van trong 02-06-2008 - 17:56

1. Hệ tọa độ xiên và ứng dụng

Còn về hệ toạ độ xiên thì em nghĩ thầy chỉ cần nêu ra lí thuyết ngắn gọn thôi,vì nó cũng gần gần giống như hệ toạ độ Decac.Đôi chút về pp này
Khái niệm: Cho điểm $O$ và 2 vector $\vec{e_1} ,\vec{e_2}$ không cùng phương,khj đó bộ $(O,\vec{e_1},\vec{e_2})$ được gọi là hệ toạ độ Anfin (hệ toạ độ xiên)
Khi đó $(O,\vec{e_1})$ được gọi là trục hoành $(Ox)$
$(O,\vec{e_2})$ được gọi là trục tung $(Oy)$
. Toạ độ của vector điểm trong hệ toạ độ xiên:
Xét $\vec{e}$ là một vector tuỳ ý,khi đó tồn tại duy nhất bộ số $x,y$ sao cho $\vec{e}=x\vec{e_1}+y\vec{e_1}$
$(x,y)$ được gọi là toạ độ của $\vec{e}$ trong $(O,\vec{e_1},\vec{e_2})$
+Về các tính chất thì nó tương tự như tính chất của các vector nên em cũng không nêu rõ phần này và phần đường thẳng trong hệ toạ độ xiên cũng thế.
Ví dụ Cho hbh ABCD,các điểm M,N thuộc cạnh AB,CD.I,J,K lần lượt là trung điểm AM,AN,MN.
CMR:BI,DJ,CK đồng quy.
Lời giải:
Xét toạ độ $(C,\vec{CB},\vec{CD})$.Ta có $C(0;0);B(1;0);D(0;1);A(1;1)$
Giả sử: $M(a;0);N(0;b)$. $\Rightarrow I( \dfrac{x-1}{a-1})=y.$
Tương tự:
$(DJ): x=\dfrac{y-1}{b-1}$
$(CK): \dfrac{x}{a}=\dfrac{y}{b}$
Gọi $E=(x_{0},y_{0})$ là giao điểm của BI,DJ.
Khi đó:
$\left\{\begin{array}{l}\dfrac{x_{0}-1}{a-1}=y_{0}\\x_{0}=\dfrac{y_{0}-1}{b-1}\end{array}\right.$
Suy ra: $\left\{\begin{array}{l}x_{0}= \dfrac{a}{a+b-ab} \\ \dfrac{b}{a+b-ab} \end{array}$
Dễ dàng kiểm nghiệm $E\in (CK)$.
Bài tập:
1>(China TST 1996) Cho tam giác ABC .Đường tròn đường kính BC cắt các cạnh AB,AC tại D va E.Gọi F,G lần lượt là hình chiếu của D,E trên BC.M là giao điểm của EF và DG.Chứng minh rằng AM vuông góc với BC
2>(đường thẳng Gauxơ)Cho tứ giac ABCD có $AB \cap CD = E; AD \cap BC=F$
CMR . Trung điểm các đoạn AC,BD,EF thẳng hàng.
3>Cho hình bình hành ABCD, các điểm X,Y,Z,T thuộc DA,AB,BC,CD sao cho. $\dfrac{AX}{AD} = \dfrac{BY}{BA} = \dfrac{CZ}{CB} = \dfrac{DT}{DC}$ .Gọi $\Delta _{1} , \Delta _{2} , \Delta _{3}$ là các đường thẳng theo thứ tự qua $A,B,C // XT,YT,ZT$.
CMR. $\Delta _{1} , \Delta _{2} , \Delta _{3}$ đồng quy.

Vì thời gian có hạn nên em cũng chỉ đưa ra những khái niệm và các bài tập cơ bản như trên thôi thầy ạ,nếu có thời gian thì em sẽ bổ sung thêm.


#177890 Câu 7 VMO 2008

Gửi bởi vo thanh van trong 29-01-2008 - 12:54

Cho tam giác ABC,trung tuyến AD.Cho đường thẳng d vuông góc với đường thẳng AD.Xét điểm M nằm trên d.Gọi E,F lần lượt là trung điểm của MB,MC.Đường thẳng đi qua E và vuông góc với d cắt đường thẳng AB ở P,đường thẳng đi qua F và vuông góc với d cắt đường thẳng AC ở Q.CMR đường thẳng đi qua M vuông góc với đường thẳng PQ luôn đi qua 1 điểm cố định ,khi điểm M di động trên đường thẳng d.


#161094 Phương pháp đặt ẩn số phụ trong giải phương trình vô tỉ

Gửi bởi vo thanh van trong 22-07-2007 - 16:37

Dạng 4 :
Nội dung phương pháp :
Cho phương trình : $\sqrt[n]{ax + b} = c(dx + e)^n + \alpha x + \beta$
Với các hệ số thỏa mãn :
$$\left\{\begin{matrix}d=ac+\alpha\\ e=bc+\beta\end{matrix}\right.$$
Cách giải :
Đặt $dy + e = \sqrt[n]{ax + b}$
Ví dụ 32: Giải phương trình:
$$\sqrt{\dfrac{4x + 9}{28} } = 7x^2 + 7$$
Lời giải:
ĐK : $x \geq - \dfrac{9}{4}$
$$PT\Leftrightarrow \sqrt{\dfrac{4x + 9}{28} } = 7(x + \dfrac{1}{2})^2 - \dfrac{7}{4}$$
- Kiểm tra: $a = \dfrac{1}{7}; b = \dfrac{9}{28} ; c = 7 ; d = 1 ; e = \dfrac{1}{2} ; \alpha = 0 ; \beta = - \dfrac{7}{4} .$
Đặt
$$y + \dfrac{1}{2} = \sqrt{\dfrac{4x + 9}{28} }$$
$$\Leftrightarrow y^2 + y + \dfrac{1}{4} = \dfrac{4x + 9}{28}\Leftrightarrow 7y^2 + 7y + \dfrac{7}{4} = x + \dfrac{9}{4}\Leftrightarrow x + \dfrac{1}{2} = 7y^2 + 7, \ \ \ (1)$$
Mặt khác : $y + \dfrac{1}{2} = 7x^2 + 7x,\ \ \ (2)$
Từ $(1)$ và $(2)$ ta có hệ :
$$\left\{\begin{matrix}x + \dfrac{1}{2} = 7y^2 + 7 \\y + \dfrac{1}{2} = 7x^2 + 7x \end{matrix}\right.$$
Đây là hệ đỗi xứng loại II đã biết cách giải .

Ví dụ 33 : Giải phương trình
$$x^2 - 6x + 3 = \sqrt{x + 3} , x \geq 3 .$$
Lời giải
$$PT \Leftrightarrow (x - 3)^2 - 6 = \sqrt{x + 3}$$
- Kiểm tra : $a = 1 ; b = 3 ; c = 1 ; d = 1 ; e = -3 ; \alpha = 0 ; \beta = - 6 .$
Đặt :
$$y - 3 = \sqrt{x + 3} \Leftrightarrow y^2 - 6y + 9 = x +3 \Leftrightarrow x - 3 = y^2 - 6y + 3, \ \ \ (1)$$
Mặt khác : $y - 3 = x^2 - 6x + 3, \ \ \ (2)$
Từ $(1)$ và $(2)$ ta có hệ :
$$\left\{\begin{matrix}x - 3 = y^2 - 6y + 3 \\ y - 3 = x^2 - 6x + 3 \end{matrix}\right.$$
Các bạn tự giải hệ trên.

Ví dụ 34: Giải phương trình:
$$\sqrt[3]{3x - 5} = 8x^3 - 36x^2 + 53x - 25$$
Lời giải :
$$PT\Leftrightarrow \sqrt[3]{3x - 5} = (2x)^3 - 3.4x^2.3 + 3.9.2x - 27 - x + 2\Leftrightarrow \sqrt[3]{3x - 5} = (2x -3)^3 - x + 2$$
- Kiểm tra :$ a = 3 ; b = - 5 ; c = 1 ; d = 2 ; e = - 3 ; \alpha = - 1 ; \beta = 2 .$
Đặt :
$$2y - 3 = \sqrt[3]{3x - 5}\Leftrightarrow (2y - 3)^3 = 3x – 5$$
$$\Leftrightarrow 8y^3 - 36y^2 + 54y - 27 = 3x - 5\Leftrightarrow 8y^3 - 36y^2 + 53y - 25 = 3x - y – 3,\ \ \ (1)$$
Mặt khác : $8x^3 - 36x^2 + 53x - 25 = 2y – 3,\ \ \ (2)$
Từ $(1)$ và $(2)$ ta có hệ :
$$\left\{\begin{matrix}8x^3 - 36x^2 + 53x - 25 = 2y - 3 \\8y^3 - 36y^2 + 53y - 25 = 3x - y - 3 \end{matrix}\right.$$
Các bạn tự giải hệ trên.


#161093 Phương pháp đặt ẩn số phụ trong giải phương trình vô tỉ

Gửi bởi vo thanh van trong 22-07-2007 - 16:35

2. Dùng ẩn phụ đưa về hệ đối xứng
Dạng 1: Phương trình dạng $x^n + b = a\sqrt[n]{ax - b}$
Cách giải: Đặt $t = \sqrt[n]{ax - b}$ ta có hệ:
$$\left\{\begin{matrix}x^n + b = at\\t^n + b = ax\end{matrix}\right.$$

Ví dụ 29: Giải phương trình $x^3 + 1 = 2\sqrt[3]{2x - 1}$
Lời giải:
Đặt: $t = \sqrt[3]{2x - 1}$ ta có:
$$t^3=2x-1\Rightarrow \left\{\begin{matrix}x^3+1=2t\\t^3+1= 2x\end{matrix}\right. \Leftrightarrow \left\{\begin{matrix}x^3+1=2t \\x^3-t^3=2(t-x)\end{matrix}\right.$$
$$\Leftrightarrow \left\{\begin{matrix}x^3+1=2t\\(x-t)(x^2+t^2+t+tx+2)=0\end{matrix}\right.\Leftrightarrow \left\{\begin{matrix}x=t\\x^3-2x+1=0\ \ \ (1)\end{matrix}\right. \vee \left\{\begin{matrix}x^3+1=2t\\x^2+t^2+tx+2=0\ \ \ (2)\end{matrix}\right.$$
$$(1)\Leftrightarrow (x-1)(x^2+x-1)=0\Leftrightarrow x=1\vee x=\dfrac{-1\pm\sqrt{5}}{2}$$
$$(2) \Leftrightarrow (t + x)^2 + x^2 + t^2 + 4 = 0, \ \ (3) $$
Phương trình $(3)$ vô nghiệm.
Vậy nghiệm của phương trình là: $x=1 ;x= \dfrac{- 1 \pm \sqrt{5} }{2}$

Dạng 2: Phương trình dạng $x = a + \sqrt{a + \sqrt{x} }$
Cách giải: Đặt $t = a + \sqrt{x}$
$$PT \Leftrightarrow \left\{ \begin{matrix}x = a + \sqrt{t} \\t = a + \sqrt{x}\end{matrix}\right.$$

Ví dụ 30: Giải phương trình $x = 2007 + \sqrt{2007 + \sqrt{x} }$
Lời giải:
ĐK: $x > 0$
Đặt: $t = 2007 + \sqrt{x},\ \ (1)$
$$PT \Leftrightarrow \left\{ \begin{matrix}x = 2007 + \sqrt{t},\ \ (2) \\t = 2007 + \sqrt{x}, \ \ (3)\end{matrix}\right.$$
Trừ từng vế của $(3)$ cho $(2)$ ta được:
$$x - t = \sqrt{t} - \sqrt{x} \Leftrightarrow (\sqrt{t} - \sqrt{x})(\sqrt{t} + \sqrt{x} + 1) = 0\Leftrightarrow x = t$$
$$(1) \Rightarrow x - \sqrt{x} - 2007 = 0\Rightarrow x = \dfrac{8030 + 2\sqrt{8029} }{4}\ \ (x > 0)$$

Dạng 3: Chọn ẩn phụ từ việc làm ngược:
Ví dụ 31: Giải phương trình $x^2 - 2x = 2\sqrt{2x - 1} $
Lời giải:
ĐK: $x \geq \dfrac{1}{2}$. Đặt$\sqrt{2x - 1} = ay + b $. Chọn $a, b$ để hệ:
$$(I) \left\{ \begin{matrix}x^2 - 2x = 2(ay + b) \\(ay + b)^2 = 2x - 1\end{matrix}\right. ,\ \ \left (x \geq \dfrac{1}{2} ; y \geq 1 \right )$$
là hệ đối xứng.
Lấy $a = 1 , b = - 1 $ta được hệ:
$$ \left\{ \begin{matrix}x^2 - 2x = 2(y - 1) \\y^2 - 2y = 2(x - 1)\end{matrix}\right.\Rightarrow \left\{ \begin{matrix}x^2 - 2x = 2(y - 1) \\x^2 - y^2 = 0\end{matrix}\right.$$
Giải hệ trên ta được: $x = y = 2 \pm \sqrt{2}$
Đối chiếu với điều kiện của hệ $(I)$ ta được nghiệm duy nhất của phương trình là: $x = 2 + \sqrt{2}$


#161092 Phương pháp đặt ẩn số phụ trong giải phương trình vô tỉ

Gửi bởi vo thanh van trong 22-07-2007 - 16:33

III. Phương pháp dùng ẩn phụ đưa về hệ
1. Dùng ẩn phụ đưa về hệ đơn giản giải bằng phép thế hoặc rút gọn theo vế .
a. Dùng một ẩn phụ .
Ví dụ 25: Giải phương trình $x^2 + \sqrt{x + 5} = 5$
Lời giải:
ĐK: $x \geq - 5$
Đặt $t = \sqrt{x + 5} , t \geq 0 $. Khi đó: $x = t^2 - 5$. Do đó ta có:
$$\left\{ \begin{array}{l} x^2 + t = 5 \\ t^2 - x = 5 \\ \end{array} \right.\Leftrightarrow \left\{ \begin{array}{l} x^2 + t = 5 \\ x^2 - t^2 + t + x =0 \\ \end{array} \right. \Leftrightarrow \left\{ \begin{array}{l} x^2 + t = 5 \\ (x + t)(x + 1 - t) = 0 \\ \end{array} \right. \Leftrightarrow \begin{array}{l} \\ \left[ \begin{array}{l} \left\{ \begin{array}{l} x^2 + t = 5 \\ x + t = 0 \\ \end{array} \right. \\ \left\{ \begin{array}{l} x^2 + t = 5 \\ x + 1 - t = 0 \\ \end{array} \right. \\ \end{array} \right. \\ \end{array}$$
Giải hệ và kiểm tra điều kiện, ta được:
$$x = \frac{{ \pm 1 - \sqrt {21} }}{2}$$

Bài toán tổng quát: Giải phương trình
$$x^2 + \sqrt{x + a} = a$$

b. Dùng 2 ẩn phụ .
Đối với phương trình dạng
$$\sqrt[m]{a + f(x)} + \sqrt[n]{b - f(x)} = c$$
Ta đặt:
$$u = \sqrt[m]{a + f(x)};v = \sqrt[n]{b - f(x)}$$
Như vậy ta có hệ:
\[\left\{ \begin{array}{l} u + v = c \\ u^m + v^n = a + b \\ \end{array} \right.\]

Ví dụ 26: Giải phương trình
$$\sqrt[4]{57 - x} + \sqrt[4]{x + 40} = 5, \ \ \ (1)$$
Lời giải:
ĐK: $ - 40 \leq x \leq 57$
Đặt $u = \sqrt[4]{57 - x} ; v = \sqrt[4]{x + 40}$
Khi đó:
$$(1) \Leftrightarrow \left\{ \begin{array}{l} u + v = 5 \\ u^4 + v^4 = 97 \\ \end{array} \right.\Leftrightarrow \left\{ \begin{array}{l} u + v = 5 \\ 2(uv)^2 - 10uv + 528 = 0 \\ \end{array} \right.\Leftrightarrow \left\{ \begin{array}{l} u + v = 5 \\ \left[ \begin{array}{l} uv = 6 \\ uv = 44 \\ \end{array} \right. \\ \end{array} \right.\Leftrightarrow \left\{ \begin{array}{l} u + v = 5 \\ uv = 6 \\ \end{array} \right.$$

Ta thu được $u = 2 ; v = 3 $hoặc $u = 3 ; v = 2$. Đến đây chỉ việc thay vào để tìm nghiệm của phương trình ban đầu .

Ví dụ 27: Giải phương trình
$$\sqrt{\sqrt{2} - 1 - x } + \sqrt[4]{x} = \dfrac{1}{\sqrt[4]{2} }$$
Lời giải:
ĐK: $0 \leq x \leq \sqrt{2} - 1 $
Đặt: $\sqrt{\sqrt{2} - 1 - x } = u ;\sqrt[4]{x} = v$ Với $0 \leq u \leq \sqrt{\sqrt{2} - 1} ; 0 \leq v \leq \sqrt[4]{\sqrt{2} - 1}$
Như vậy ta được hệ:
$$\left\{\begin{array}{l}u+v=\dfrac{1}{\sqrt[4]{2}}\\u^2+v^4=\sqrt{2}-1.\end{array}\right.\Rightarrow \left\{\begin{array}{l}u=\dfrac{1}{\sqrt[4]{2}}-v\\ \left (\dfrac{1}{\sqrt[4]{2}}-v \right)^2+v^4=\sqrt{2}-1\end{array}\right.$$

Giải $(1)$:
$$(1)\Rightarrow (v^2+1)^2-\left (\dfrac{1}{\sqrt[4]{2}}+v\right )^2 = 0\Rightarrow v^2-v+1-\dfrac{1}{\sqrt[4]{2}}=0\Rightarrow v_{1,2}=\dfrac{1\pm\sqrt{\dfrac{4}{\sqrt[4]{2}}-3}}{2},\ \ (v_{1,2} > 0)$$
Vậy $v_{1,2}$ (thỏa mãn điều kiện) chính là 2 nghiệm của phương trình đã cho .

Ví dụ 28: Giải phương trình:
$$\sqrt{\dfrac{7}{4}\sqrt{x} - 1 + x^2} = (1 - \sqrt{x})^2$$
Lời giải:
Đặt: $y = \sqrt{x} , y \geq 0;z = 1 - \sqrt{x}$. Ta có:
$$ \Leftrightarrow \left\{ \begin{array}{l}y+z=1, \ \ \ (1)\\uv=6 \end{array} \right.\Leftrightarrow \left\{ \begin{gathered} y + z = 1 \\ {y^4} - {z^4} = \frac{7}{4}\sqrt x - 1, \ \ (2)\end{gathered} \right. $$
Thế $(1)$ vào $(2)$ ta có
$$y^4 - (1 - y)^4 = \dfrac{7}{4}y - 1\Rightarrow 4y(y - \dfrac{3}{4})^2 = 0\Leftrightarrow \left[ \begin{array}{l}y=0\\y=\frac{3}{4}\end{array} \right. \Leftrightarrow \left[ \begin{array}{l}x=0\\x=\frac{9}{{16}}\end{array} \right.$$


#161091 Phương pháp đặt ẩn số phụ trong giải phương trình vô tỉ

Gửi bởi vo thanh van trong 22-07-2007 - 16:30

3. Dùng 3 ẩn phụ .
Ví dụ 23: Giải phương trình
$$\sqrt[3]{7x + 1} - \sqrt[3]{x^2 - x - 8} + \sqrt[3]{x^2 - 8x + 1} = 2$$
Lời giải:
Đặt $ a = \sqrt[3]{7x + 1} , b = - \sqrt[3]{x^2 - x - 8} , c = \sqrt[3]{x^2 - 8x + 1}$, ta có:
$$ a + b + c = 2$$
$$ a^3 + b^3 +c^3 = (7x + 1) - (x^2 - x - 8) + (x^2 - 8x - 1) = 8,\ \ \ (1)$$
Mặt khác: $ (a + b +c)^3 = 8,\ \ \ (2)$
Từ $(1)$ và $(2)$ ta có:
$$ (a + b + c)^3 - (a^3 + b^3 +c^3) = 3(a + b)(b + c)(c + a)$$
Nên:
$$ (a + b)(b + c)(c + a) = 0 \Leftrightarrow \left[ \begin{array}{l} a = - b \\ b = - c \\ c = - a \\ \end{array} \right.$$
Từ đó dễ dàng tìm ra 4 nghiệm của phương trình: $ S = {- 1 ; 0 ; 1 ; 9}$

Ví dụ 24: Giải phương trình
$$\sqrt[3]{3x + 1} + \sqrt[3]{5 - x} + \sqrt[3]{2x - 9} - \sqrt[3]{4x - 3} = 0, \ \ \ (1)$$
Lời giải:
Đặt $ a = \sqrt[3]{3x + 1} ; b = \sqrt[3]{5 - x} ; c = \sqrt[3]{2x - 9}$ Suy ra:
$$ a^3 + b^3 + c^3 = 4x - 3$$
khi đó từ $(1)$ ta có:
$$ (a + b + c)^3 = (a^3 + b^3 +c^3) \Leftrightarrow (a + b)(b + c)(c + a) = 0$$
Giải như ví dụ 23 suy ra được 3 nghiệm của phương trình: $ x = -3 ; x = 4 ; x = \dfrac{8}{5}$


#161090 Phương pháp đặt ẩn số phụ trong giải phương trình vô tỉ

Gửi bởi vo thanh van trong 22-07-2007 - 16:28

2. Dùng 2 ẩn phụ .
Ví dụ 19: Giải phương trình
$$\sqrt{4x^2 + 5x + 1} - 2\sqrt{x^2 - x + 1} = 9x - 3$$
Lời giải
Đặt $ a = \sqrt{4x^2 + 5x + 1} , b = 2\sqrt{x^2 - x + 1}$
$$\Rightarrow a^2 - b^2 = 9x – 3 \Rightarrow a - b = a^2 - b^2 \Leftrightarrow (a - b)(a + b - 1) = 0$$
*$ a - b = 0 \Rightarrow x = \dfrac{1}{3}$
*$ a + b - 1 = 0 \Rightarrow \left\{ \begin{array}{l} a - b = 9x - 3 \\ 2a = 9x - 2 \\ \end{array} \right. \Rightarrow \left[ \begin{array}{l} x = 0 \\ x = \frac{{56}}{{65}} \\ \end{array} \right.$

Ví dụ 20: Giải phương trình
$$2(x^2 - 3x + 2) = 3\sqrt{x^3 + 8},\ \ \ (1)$$
Lời giải:
ĐK : $ - 2 \leq x \leq 1$ hoặc$ x \geq 2$
Đặt $ u = \sqrt{x^2 - 2x + 4} , v = \sqrt{x + 2}$ ta có :
$$ u^2 - v^2 = x^2 - 3x + 2 .$$
$(1)$ trở thành :
$$ 2(u^2 - v^2) = 3uv \Leftrightarrow (2u + v)(u - 2v) = 0 \Leftrightarrow u = 2v$$
(Do $ 2u + v > 0$)
Để tìm $x$, ta giải :
$$\sqrt{x^2 - 2x + 4} = 2 \sqrt{x + 2} \Leftrightarrow x^2 - 6x - 4 = 0 \Leftrightarrow x = 3 \pm \sqrt{13}$$
Kết hợp với điều kiện, phương trình $(1)$ có 2 nghiệm : $ x = 3 \pm \sqrt{13}$

Ví dụ 21: Giải phương trình
$$\sqrt{5x^2 - 14x + 9} - \sqrt{x^2 - x - 20} = 5\sqrt{x + 1}, \ \ (1)$$
Lời giải:
ĐK : $ x \geq 5$
Chuyển vế rồi bình phương hai vế, ta được:
$$ (x + 1)(5x + 9) = x^2 + 24x + 5 + 10\sqrt{(x + 4)(x - 5)(x + 1)}$$
$\Leftrightarrow 2(x^2 - 4x - 5) + 3(x + 4) - 5\sqrt{(x^2 - 4x - 5)(x + 4)} = 0,\ \ \ (2)$
Đặt $ u = \sqrt{(x^2 - 4x - 5)}$ và $ v = \sqrt{x + 4} , u,v \geq 0 .$ Thì:
$$(2)\Leftrightarrow 2u^2 + 3v^2 - 5uv = 0 \Leftrightarrow (u - v)(2u - 3v) = 0$$
* $ u = v$ ta có :$ x^2 - 5x - 9 = 0$
* $ 2u = 3v$ ta có : $ 4x^2 - 25x - 56 = 0$
Giải ra ta được 2 nghiệm thỏa mãn: $ x = \dfrac{5 + \sqrt{61} }{2} , x = 8$

Ví dụ 22: Giải phương trình
$$\sqrt{x} + \sqrt[4]{x(1 - x)^2} + \sqrt[4]{(1 - x)^3} = \sqrt{1 - x} + \sqrt[4]{x^3} + \sqrt[4]{x^2(1 - x)}$$
Lời giải:
ĐK : $ 0 \leq x \leq 1$
Đặt: $\left\{ \begin{array}{l} u = \sqrt[4]{x} \\ v = \sqrt[4]{{1 - x}} \\ \end{array} \right. \Rightarrow \left\{ \begin{array}{l} u \ge 0 \\ v \ge 0 \\ u^4 + v^4 = 1 \\ \end{array} \right.$
Từ phương trình ta được :
$$ u^2 + uv^2 + v^3 = v^2 + u^3 + u^2v$$
$\Leftrightarrow (u - v)(u + v)(1 - u - v) = 0\Leftrightarrow (u - v)(1 - u - v) = 0 $( Do $ u + v > 0$)

từ đó ta giải ra được các nghiệm :$ x = 0 , x = \dfrac{1}{2} , x = 1 $


#161085 Phương pháp đặt ẩn số phụ trong giải phương trình vô tỉ

Gửi bởi vo thanh van trong 22-07-2007 - 16:09

III. Phương pháp dùng ẩn phụ đưa về dạng tích
1. Dùng một ẩn phụ
Ví dụ 15 Giải phương trình
$$x^2 + \sqrt{x + \dfrac{3}{2} } = \dfrac{9}{4},\,\,(1)$$
Lời giải
ĐK : $x \geq - \dfrac{3}{2} $.
Đặt $\sqrt{x + \dfrac{3}{2} } = t , t \geq 0 $ phương trình $(1)$ trở thành :
$$(t^2 - \dfrac{3}{2})^2 = \dfrac{9}{4} – t \Leftrightarrow t(t^3 - 3t + 1) = 0 \Leftrightarrow
\left[ \begin{array}{l} t = 0 \\ t^3 - 3t + 1 = 0,\,\, (2) \\ \end{array} \right.$$
$(2)$ giải đựoc bằng cách áp dụng phương pháp I :
Đặt $x = 2cost , t \in (0 ; \pi)$ để đưa về dạng : $cos3t = - \dfrac{1}{2}$

Tổng quát: Giải phương trình
$$x^2 + \sqrt{x + a} = a^2$$
Với $a$ là hắng số cho trước .

Ví dụ 16: Giải phương trình:
$$x^3 - 3x^2 + 2\sqrt{(x + 2)^3} =6x,\,\, (1)$$
Lời giải:
ĐK : $x \geq - 2$
Viết lại $(1)$ dưới dạng :
$$x^3 - 3x(x + 2) + 2 \sqrt{(x + 2)^3} = 0,\,\,(2)$$
Đặt $t = \sqrt{x + 2} \geq 0 $. Khi đó $(2)$ trở thành :
$$x^3 - 3xt^2 + 2t^3 \Leftrightarrow (x - t)^2(x + 2t) = 0$$
Do vậy $x = t$ hoặc $x = -2t$
*$x = t $. Ta có :
$$x = \sqrt{x + 2}\Leftrightarrow \left\{ \begin{array}{l} x \ge 0 \\ x^2 - x - 2 = 0 \\ \end{array} \right. \Leftrightarrow x = 2$$
*$ x = -2t$ . Ta có :
$$x = - \sqrt{x + 2}\left\{ \begin{array}{l} x \leq 0 \\ x^2 - 4x - 8 = 0 \\ \end{array} \right. \Leftrightarrow x = 2 - 2\sqrt{3}$$
Vậy phương trình đã cho có 2 nghiệm : $x = 2 , x = 2 - 2 \sqrt{3}$

Ví dụ 17: Giải phương trình:
$$x + \sqrt{5 + \sqrt{x - 1} } = 0$$
Lời giải:
ĐK : $x \in [1 ; 6],\,\,\ (1)$
Đặt $t = \sqrt{x - 1} , t \geq 0,\ \ \ (2)$ phương trình đã cho trở thành :
$$t^2 + \sqrt{5 + t} = 5 ,\ \ \ (3)$$
$$\Leftrightarrow t^4 - 10t^2 - t + 20 = 0 \Leftrightarrow (t^2 + t -4)(t^2 - t - 5) = 0$$
Đối chiếu với hai điều kiện $(1)$ và $(2)$ thay vào và giải ra :
$$x = \dfrac{11 - \sqrt{17} }{2}$$

Ví dụ 18: Giải phương trình:
$$x = \left (2006 + \sqrt{x} \right )\left (1 - \sqrt{1 - \sqrt{x}} \right )$$
Lời giải:
ĐK : $x \in [0 ; 1],\ \ \ \ (1)$
Đặt $t = \sqrt{1 - \sqrt{x} }\Rightarrow 0 \leq t \leq 1$. Khi đó:
$$\sqrt{x} = 1 - t^2 , x = (1 - t^2)^2 $$
phương trình đã cho trở thành :
$$(1 - t^2)^2 = (2006 + 1 - t^2)(1 - t)^2$$
$$\Leftrightarrow (1 - t)^2(1 + t)^2 = (2007 - t^2)(1 - t)^2 \Leftrightarrow 2(1 - t)^2(t^2 + t - 1003)$$
Vì $0 \leq t \leq 1$ nên: $t^2 + t - 1003 < 0$
Do đó phương trình tương đương với :
$$t - 1 = 0 \Leftrightarrow t = 1$$
Do vậy $x = 0$ (thỏa $(1)$)